Ask your own question, for FREE!
Mathematics 31 Online
OpenStudy (anonymous):

Limit as n approaches ∞ of n(√(n^2 +1) -n) using Lo Hospital's rule. I tried using Lo Hospital's rule on it in the form (√(n^2 +1) -n)/n^-1 but it just got more complicated... Not sure what to do. Probably missing a sequence theorem.

OpenStudy (anonymous):

you gotta make it a rational function first

OpenStudy (anonymous):

Here's the part where I admit that despite taking AP Calc I don't know what a rational funciton is

OpenStudy (anonymous):

*function

OpenStudy (anonymous):

A function over another function

OpenStudy (anonymous):

do you got this right|dw:1452403747258:dw|

Can't find your answer? Make a FREE account and ask your own questions, OR help others and earn volunteer hours!

Join our real-time social learning platform and learn together with your friends!
Latest Questions
YoungBlood: STOP THE YELLOW NAME TAG SLANDER!
4 hours ago 11 Replies 2 Medals
Bubblezz: Art for @euphoriiic
7 hours ago 23 Replies 3 Medals
ilovemybf: i need more drawing ideas
8 hours ago 15 Replies 1 Medal
toga: what is a Mayuri
12 hours ago 3 Replies 1 Medal
Midnight97: Here is a beat I made let me know what y'all think
12 hours ago 24 Replies 2 Medals
toga: who thinks that there should be more titles
13 hours ago 5 Replies 0 Medals
Can't find your answer? Make a FREE account and ask your own questions, OR help others and earn volunteer hours!

Join our real-time social learning platform and learn together with your friends!